i need to know whether it is the minimum or maximum function and what that number is as well as where x axis intercepts the quadratic graph . i need to know if there are any roots in the quadratic graph, and i need to know when y=1 , the value of x could be x= or x=

Answers

Answer 1

Answer:

The minimum value of this quadratic function is -4.

At the point (-4, 0), the x-axis intercepts with the quadratic graph.

There is 1 root of this quadratic equation.

Approximately, when y=1, the x value could be -5 or -3.


Related Questions

Find the area of this circle

Answers

Answer:

314ft²

Step-by-step explanation:

A = 3.14(10²)

A = 3.14 x 100

A = 314ft²

Answer:

Area is 314 ft²

Step-by-step explanation:

[tex]A = \pi {r}^{2} [/tex]

r is radius,

[tex]r = \frac{diameter}{2} = \frac{20}{2} = 10 \: ft[/tex]

therefore:

[tex]A = 3.14 \times {10}^{2} \\ A = 3.14 \times 100 \\ A = 314 \: {ft}^{2} [/tex]

You go to the grocery store to buy milk and some eggs. Milk costs $3.67 and eggs cost $2.40. You want to make sure you have enough money. Estimate the cost of these two items to the nearest ones place.

Answers

Answer:

Step-by-step explanation:

Let's start by round 3.67. We look at the key digit which is 6 in this case. Since 6 is greater than 5, 3.67 will round up to 4.

The key digit in 2.40 is 4. 4 is not greater than 5, so we round down to 2.

2+4=6

Therefore, an estimate of these two items to the nearest ones place is $6.

Answer:

$2 for the eggs and $4 for the milk

Step-by-step explanation:

Since the eggs cost $2.40 and after the 2 you can see that there is a 4, " 5 or more we add one more, 4 or less just ignore", so it stays $2. However the milk costs $3.67, and here you can see that after the 3 there is a 4, so since it's more than 5 we change the estimated amount to $4.

I hope you understood, and I hope the answer is right :)

7 + (-24) / 8 + 4 (-2) =

Answers

Answer: 0

Step-by-step explanation:

7+(-24)/8+4(-2 )= 7-24/8+4(-2) = -17/8-8 = -17/0 = 0

The nth term of a geometric sequence is 4*3n-1. Find the first and the 10th term

Answers

Answer:

Step-by-step explanations

I don’t know

GH = 4x - 1, and DH = 8. Find x.
Help

Answers

x=197 is the answer for your question

Answer:

x=4.25

------------------------------------------

8+8=4x-1

16=4x-1

4x=16+1

4x=17

x= 17/4

I have no idea if i am correct just a guesstimate

Have a good day

The table shows the times for the 100-meter dash. What is the
difference between the men'stime and the women's time?
Write your answer in both decimal and fraction form.
Times for 100-meter Dash
Men's winner
9.58
2009
Women's winner
10.49
1988

Answers

Answer:

0.91 (make sure to include unit of measurement)

Step-by-step explanation:

10.49 - 9.58 = 0.91

express 3x-x^2 as m-(x-n)^2. Ill give brainliest to correct answer!

Answers

Answer:

(i) 2x+3y=9.3

5

ˉ

⇒2x+3y−9.3

5

ˉ

Comparing the equation with ax+by+c=0

a=2,b=3,c=−9.3

5

ˉ

Answer:

[tex]\frac{9}{4}[/tex] - (x - [tex]\frac{3}{2}[/tex] )²

Step-by-step explanation:

Given

3x - x² ← factor out - 1 from each term

= - (x² - 3x)

Using the method of completing the square

add/subtract ( half the coefficient of the x- term)² to x² - 3x

= - (x² + 2(- [tex]\frac{3}{2}[/tex])x + [tex]\frac{9}{4}[/tex] - [tex]\frac{9}{4}[/tex] )

= - (x - [tex]\frac{3}{2}[/tex] )² - 1(- [tex]\frac{9}{4}[/tex] )

= [tex]\frac{9}{4}[/tex] - (x - [tex]\frac{3}{2}[/tex] )²

What is the slope of a line parallel to a line with equation -4y + 7x = 20?

Answers

Answer:

[tex]{ \underline{ \sf{slope \: is \: \frac{7}{4} }}}[/tex]

Step-by-step explanation:

Parallel lines have same slopes.

General equation of line:

[tex]y = mx + c[/tex]

the line given:

[tex] - 4y + 7x = 20 \\ 4y = 7x - 20 \\ y = \frac{7}{4} x - 5[/tex]

Complete the table. Round each number to given place. Number 155,999 to nearest hundred

Answers

Answer:

156,000

Step-by-step explanation:

Answer:

156,000

Step-by-step explanation:

Note the hundreds place value (underlined and bold):

155,999

Look at the digit value directly next to the given place value (tens place value). It is a 9. Because 9 is greater then 5*, you will round up:

155,999 rounded to the nearest hundreds place value is 156,000.

* Note when rounding:

If the digit to the right of the one you are finding is:

- Less than 5: Round down.

- 5 or greater: Round up.

~

Given IQ scores are approximately normally distributed with a mean of 100 and a standard deviation of 15 , the proportion of people with s above 130 is:

Answers

Given the assumption of a normal distribution with a mean of 100 and a standard deviation of 15, a score of 130 represents a z-score of 2. In different mostly older handbooks of descriptive statistics you can find a ‘from z to percentile-table’. Nowadays it’s more easy to use the computer. Using R, a free and open source statistical environment (www.r-project.org), you can use the command pnorm (130, 100, 15) and it will give 0.9772. Because yo want the proportion above that score you use 1-pnorm (130, 100, 15). Another way of writing in R and with for example 3 IQ-scores:

perc = pnorm (c (70, 100, 130), 100, 15)

(1 - perc)

gives you the above-proportion of the IQ-scores of respectively 70, 100 and 130.

Detailed answer if possible I look to try and understand how to do this. So there is no need for a rushed answer just a detailed one so I can know how to do problems like these.

Answers

Answer:

yes

Step-by-step explanation:

solve each of the inequalities illustrating each solution on a number line
a)x-3 ≥7
b)-2y+4> 3

Answers

Answer:

a) [tex]x \geqslant 10[/tex][tex]y < - \frac{ - 1}{2} [/tex]

Step-by-step explanation:

mark me as brainliest ❤️

Please help it’s due today


Mark brainliest!!!


1.47-1.56

Answers

Answer:

0.09

Step-by-step explanation:

Find the output, k, when the input, t, is -7.

k= 10t - 19

k=

Answers

Answer:

Here is the answer . hope this helps.

Is 9pi rational or irrational

Answers

Answer:

irrational

Step-by-step explanation:

every number that times with pi are irrational, only zero gives a rational number

Type the correct answer in the box. Round your answer to the nearest hour. A scientist running an experiment starts with 100 bacteria cells. These bacteria double their population every 15 hours. Find how long it takes for the bacteria cells to increase to 300. Use the formula , where is the original number of bacteria cells, is the number after t hours, and d is the time taken to double the number. It takes_____ hours for the number of bacteria to increase to 300.

Answers

Answer:

It takes 45 hours for the bacteria to triple the initial amount

Answer:

45 HOURS IS CORRECT

Step-by-step explanation:

George has $15. He tries to buy a movie ticket ($9.00), a pretzel ($2.65), a drink ($1.35), and two veggie cups ($1.74 each) but he does not have enough money. Look at the numerical expression George used: 15-9+2.65+1.35+2(1.74)
Why does George need to fix the expression?

Answers

George needs to fix the expression because if you did the equation as is, it wouldn't subtract the added value from 15.

George always requires $1.48.

How to estimate the value of the expression?

The order of operations exists as a law that tells the correct sequence of measures for considering a math expression. We can determine the mandate utilizing PEMDAS: Parentheses, Exponents, Multiplication, and Division (from left to right), Addition, and Subtraction (from left to right).

Given: George tries to purchase a movie ticket ($9.00), a pretzel ($2.65), a drink ($1.35), and two veggie cups ($1.74 each)

9 + 2.65 + 1.35 + 1.74 + 1.74 = 16.48

Subtract 15 to 16.48

15 - 16.48 = 1.48

George always needs $1.48.

Therefore, the proper answer is $1.48.

To learn more about the PEMDAS refer to:

https://brainly.com/question/24605146

#SPJ2

Plss help me! Brainlist! Pls!!! I’m so confused!!!!

Answers

3^ (1-2x)=243 and ya that’s how you solve it if u don’t understand let me know

Compare with >, < 12.6 and 12.3​

Answers

Answer:

12.6>12.3

Step-by-step explanation:

Does this scatter plot show a positive trend, a negative trend, or no trend?

Answers

Answer:

This shows no trend.

Step-by-step explanation:

Answer:

No Trend

Step-by-step explanation:

The scatter plot is everywhere and there is no pattern within it therefore it has no trend.

Hope it helps

Answer all questions olve the following logarithmic ec 2 log10 x + log103 = log10 75
[tex]2 log_{10}x + log_{10}(3) = log_{10}(75) [/tex]






Answers

Step-by-step explanation:

We can rewrite the left-hand side as

[tex]2\log{x} + \log{3} = \log{3x^2}[/tex]

Here, we used the following properties of logarithms:

[tex]\log{x^a} = a\log{x}[/tex]

[tex]\log{ab} = \log{a} + \log{b}[/tex]

We can then write

[tex] \log{3x^2} = \log{75} \Rightarrow 3x^2 = 75[/tex]

or

[tex]x = 5[/tex]

Perform the indicated operation.
(7x + 2x2 - 6x - 4) - (2x+3x2 - 3x + 11)
(7x + 2x2 - 5x - 4) - (2x + 3x² 3x + 11) =
(Simplify your answer. Do not factor.)

Answers

Step-by-step explanation:

1st.

(7x+2×2-6x-4) - (2x+3×2-3x+11)

(7x+4-6x-4)-(2x+6-3x+11)

collect like terms

(7x-6x-4+4)-(2x-3x+11+6)

(x+0)-(-x+17)

multiply - by the bracket

x+0+x-17

collect like terms

x+x+0-17

2x-17=0

2x=17

divide both sides by the coefficient of x

2x/2=17/2

x=17/2

How many centimeters are in 5.2 inches?

[1 inch = 2.5 cm]

13 cm
11 cm
7.30 cm
1.92 cm

Answers

Step-by-step explanation:

Since,

1 inch=2.5 cm

Therefore,

5.2 inches=2.5×5.2 cm=13 cm

I hope it helped U

stay safe stay happy

Answer:

1inc=2.5cm 5.2inches=?5.2×2.552/10 × 25/10when u reduce u get260/20 answer is13cm

6(10+5)=6•10+6•5 true or false

Answers

Answer:

true, I hope this will help.

Answer:

[tex]\boxed{\underline{\bf \: True}}[/tex]

Step-by-step explanation:

[tex] \sf \: 6(10 + 5) = 6•10 + 6•5 [/tex]

Check with LHS = RHS rule method.

LHS

[tex]\sf6(10 + 5) \\ \sf = 6(15) \\ \sf = 6 \times 15 \\ \bf = 90[/tex]

RHS

[tex]\sf6•10 + 6•5 \\ \sf = 60 + 30 \\ \bf= 90[/tex]

We can see that both LHS & RHS give [tex]\boxed{\underline{\bf \: 90}}[/tex]as the solution. LHS = RHS. Hence proved.

Answer -> [tex]\boxed{\underline{\bf \: True}}[/tex]

_______________

Hope it helps.

RainbowSalt2222

Wyatt translates both lines 6 units
up and 2 units to the left. Which
of the following must be true?
A. After the translation, line f will not
intersect line g.
B. After the translation, line f will be
the same line as g.
C. After the reflection, line f will not
intersect the y-axis.
D. After the reflection, line g will not
intersect the x-axis.

Answers

The answer is A.

A translation is a rigid motion, so the shape and length of the figure will not be changed in any way, only the position. C and D can’t be the answer because there was no reflection, only a translation. And it can’t be B because both lines are being translated the same distance.

10% + 20% + 70% divided by 250 milliion

Answers

Answer: 8.16e-10

I think idrk          

Answer:

[tex] \frac{10\% +20\% + 70\% }{250 \: million} \\ \\ \\ \frac{ \frac{10}{100} + \frac{20}{100} + \frac{70}{100} }{250 \times 10 ^{6} } = \frac{ \frac{100}{100} }{250 \times {10}^{6} } \\ \\ \frac{1}{250 \times {10}^{6} } = 4 \times {10}^{ - 9} [/tex]

I hope I helped you^_^

The product of 2 and a number k, subtracted from 17

A. 2+k+17

B. 2k+17

c. 17-2k

D. 2k-17​

Answers

hi <3

product means multiplication, therefore we know there has to be a '2k' and we can eliminate option A

also, there is a mention of subtraction, so we can also eliminate option B

now we are left with C and D

it is stated that it is being subtracted FROM 17, therefore the 17 would have to come first

in short, your answer is option C

hope this helps :)

The product of 2 and a number k, subtracted from 17 is the expression: c. 17-2k.

What is an expression?

An expression is any mathematical statement which consists of numbers, variables and an arithmetic operation between them. In mathematics, an expression or mathematical expression is a finite combination of symbols that is well-formed according to rules that depend on the context.

here, we have,

given that,

The product of 2 and a number k, subtracted from 17

we have to find the expression for given statement.

let the number be x

now, we have,

The product of 2 and a number k,

i.e. 2*k = 2k

then,  subtracted from 17

i.e. 17 - 2k

so, we get,

x = 17 - 2k

Hence, The product of 2 and a number k, subtracted from 17 is the expression: c. 17-2k.

To learn more on Expression click:

brainly.com/question/14083225

#SPJ2

Find the simple interest.
Principal
Rate 11% principle 9000 time1 1/2 years what's the amount of simple interest earned is

Answers

Answer:

Here is your answer. Hope this helps you!

Answer:

Rate(R)= 11%

Times(T)=11/2

Principle(P)= 9000

We know that,

SI = P×T×R÷100

= 9000×11/2×11÷100

=9000×5.5×11÷100

= 544500÷100

=5445

Hence, the required simple interest is 5445.

Ans

What is the least three-digit whole number that has exactly five positive factors?

Answers

The number 100 and that’s the only one I can think of

There is no three-digit whole number that has exactly five positive factors.

The prime factorization of a number with five factors will be of the form p⁴ or p²q, where p and q are distinct prime numbers.

p⁴:

The smallest prime number greater than or equal to 10 is 11.

So, let's consider 11⁴:

11⁴ = 14641 (which is a five-digit number)

p²q:

To minimize the number, we need the smallest prime numbers possible. The two smallest prime numbers greater than 10 are 11 and 13.

Let's consider the product of these two primes:

11² × 13 = 1573 (a four-digit number)

To learn more on Number system click:

https://brainly.com/question/22046046

#SPJ2

log square root 27 ÷ log 81

Answers

I think 0.33333333333333 log square root, I actually don’t know hope that helps
Other Questions
ILL MARK BRAINIEST IF YOU DO THIS RIGHT!!! Complete both Part A and Part B.Part A:Jason buys a pair of pants for $28.97, a top for $12.82 and a scarf for $9.18. He has a gift card worth $75. If the sales tax on the purchase is $3.19, what is the total of his purchase?Part B:Will he have enough money left to buy a hat for $27.15?Explain your answer. how do you simplify this question x^3z^7/x^5z What is the complete subject of this sentence?Harley led the pack when she ran the 3,000 meter run. find the domain of y= 3x+3 Please help........... One number is 8 more than the other number. Twice the smaller number is 7 more than the larger number. Write the equation to represent this scenario. How does deforestation affect the amount of water in rivers and the air we breathe? ( x+y+z ) + 2( x+y+z ) +1 Mr Guru bought a second hand bike for rupees 125200 he Spent rupees 5302 repair it at what price should he sell it to gain 5% on his total investment Renaldo can rollerblade 9.7 miles per hour. at this rate, how far will he rollerblade in 0.75 hours? find the last two digits of 2^2016 2.2. Explain any SIX Human Rights that were designed specifically to promote respect for diversity difference between observation and inference I like eating bacon and eggs D OR I (2t + 1) (t 9) = -52 8tShow work ion get it Please help ASAP!!!! Una palabra aguda la podemos encontrar en la items A pjaroB mustrameloC amistad D lugar Discuss four contributing factors that may lead to an increase of learners abusing substance in schools. Hello! Can someone please answer the photo? I honestly forgot about Prime Factorization... No links please and thank you!